LSAT and Law School Admissions Forum

Get expert LSAT preparation and law school admissions advice from PowerScore Test Preparation.

 SherryZ
  • Posts: 124
  • Joined: Oct 06, 2013
|
#12231
Thank you so much for your help! It means a great deal to me!

Oct 1999 LSAT, Sec 4, Q9:

I have no idea what I was doing when I confronted my question (my brain shut down :( )

My understandings to the stimulus are:
1. The oil price is up + gov promotes energy conservation :arrow: Many ppl use gas instead :arrow: Use of oil is down by 40% (1970-Now)
2. Use gas :arrow: must install equipment
3. The author's prediction: Unlikely to switch back to use oil in the near future

Question stem is: The PREDICTION would be called into question if IN THE LAST FEW YEARS...
It means that we have to find an answer that would make the prediction become LIKELY to switch back to use oil in the near future.

My own pre-phrases are:
1. The oil price went down, even cheaper than gas price. So ppl wants to use oil again
2. The gas equipment price went up, which made gas usage not be a good deal. So ppl wants to use oil again.

I chose B, but the correct answer is D. Could you explain this question and answer choices A, B, C, D and E? Because this question bothers me seriously :(

Thank you very much! Looking forward to your reply ;)

---Sherry
User avatar
 Dave Killoran
PowerScore Staff
  • PowerScore Staff
  • Posts: 5852
  • Joined: Mar 25, 2011
|
#12255
Hi Sherry,

In your understanding of the stimulus, one thing I would change is the tense of what occurred. The author notes that "Because switching to natural gas involved investing in equipment..." (italics added), and this means that all those houses are already switched over. They've made the investment already, and because they made the investment, they aren't likely to make a switch back to oil because that would mean new equipment.

This has an effect because it means that part 2 of your prephrase doesn't apply. You said, "2. The gas equipment price went up, which made gas usage not be a good deal. So ppl wants to use oil again." But, these homes already have gas equipment, so if gas equipment prices go up, it really doesn't' affect them. What you needed here in part 2 of your prephrase is that, "2. The oil equipment price went way down, which made switching not such a big deal." That point combined with your part 1, gives you answer choice (D) (which is the only answer to address oil equipment, by the way).

From that perspective, take a look again at the other answer choices. Answers like (A) and (C)--which you probably thought initially looked good--contain the gas equipment falling sharply, which now we can see doesn't help call the prediction into question. An answer like (B) also trades on gas equipment, so again it misses the mark. And (E) fails to address the equipment issue at all.

Thanks!
 eober
  • Posts: 107
  • Joined: Jul 24, 2014
|
#15981
Hi,

In this question the author is talking about a decrease in oil usage (by 40%) and an increase in natural gas usage. However, this switch included additional costs due to investing in equipment necessary for natural gas usage. If I understood this part correctly, how can a fall in the cost and price of oil equipment not cause a switch back to oil? I thought D was supporting the argument instead of weakening it.

Thank you for the clarification!
 Emily Haney-Caron
PowerScore Staff
  • PowerScore Staff
  • Posts: 577
  • Joined: Jan 12, 2012
|
#15991
Hi eober,

The conclusion here is that there will not be a switch back to oil (presumably because it would be too expensive, since people already invested in the natural gas equipment). D calls that into question by making it clear that actually, switching back to oil would be very cheap, and maybe even save money in the long run.
 reop6780
  • Posts: 265
  • Joined: Jul 27, 2013
|
#16083
This is weaken question. The correct answer is D while I chose B.

Since the conclusion of the stimuli is that a significant switch back to oil in the near future is unlikely, I thought providing good reason to use oil again such as drop in the price of oil would be a correct answer.

Answer D suggests drop in the price of oil so that this would encourage people to use oil again in lieu of natural gas. I admit that this is a good answer that I easily passed.

Still, answer B seems to do similar job to make oil look more appealing to customers with the price of natural gas sharply increasing.

Can you explaine why only answer D is correct?

Thank you, powerscore!
 Lucas Moreau
PowerScore Staff
  • PowerScore Staff
  • Posts: 216
  • Joined: Dec 13, 2012
|
#16154
Hello, reop,

Ah, Weaken questions. The answer choice that most weakens the link between premises and conclusion is the only right one. Most is my second least favorite word on the LSAT. (My least favorite is "EXCEPT." ;) )

Also, I believe you may have slightly misread answer choice B. It does not say that the price of natural gas itself is increasing, but rather the price of the equipment to heat homes with natural gas.

Since they already invested in the equipment, the price of the equipment no longer matters (the money only needs to be spent once) compared to the price of the gas itself. With that, I believe it is far more clear why answer choice D is correct. :-D

Hope that helps,
Lucas Moreau

Get the most out of your LSAT Prep Plus subscription.

Analyze and track your performance with our Testing and Analytics Package.